Đến nội dung

Minhnguyenthe333 nội dung

Có 788 mục bởi Minhnguyenthe333 (Tìm giới hạn từ 28-04-2020)



Sắp theo                Sắp xếp  

#609474 Ảnh thành viên

Đã gửi bởi Minhnguyenthe333 on 17-01-2016 - 17:18 trong Góc giao lưu

Hay không


rất hay :))



#609379 Ảnh thành viên

Đã gửi bởi Minhnguyenthe333 on 17-01-2016 - 07:51 trong Góc giao lưu

Lâu quá up hình mắc công mọi người quên mặt mình  :D 
 

attachicon.gifCapture.JPG

Em cũng đang đọc cuốn sách đó: 17 phương trình thay đổi thế giới của Ian Stewart :))



#609513 Ảnh thành viên

Đã gửi bởi Minhnguyenthe333 on 17-01-2016 - 20:33 trong Góc giao lưu

bao nhiêu tiền quyển ấy


190.000đ



#593077 $\boxed{{Topic}}$ Ôn thi học sinh giỏi lớp...

Đã gửi bởi Minhnguyenthe333 on 10-10-2015 - 18:19 trong Chuyên đề toán THCS

Bài 185:
a) Tìm số dư của phép chia $S_{n}=1^{n}+2^{n}+3^{n}+4^{n}$ cho 4 với $n\in N$

Với $n=1$ thì $S_n\equiv 2(mod4)$
Với $n=0$ thì $S_n\vdots 4$
Với $n=2k+1(k\in N,k>0)$.Dễ thấy $1\equiv -3,1(mod4)$ và $3^{n}\equiv 3(mod4)$
$\Rightarrow S_n\equiv 0(mod4)$ khi $n>1$ và $n$ lẻ
Với $n=2k(k>0,k\in N)$.Tương tự như trên mà $3^n\equiv 1(mod 4)$
$\Rightarrow S_n\equiv 2(mod4)$ khi $n$ là số chẵn lớn hơn 0



#590570 $\boxed{{Topic}}$ Ôn thi học sinh giỏi lớp...

Đã gửi bởi Minhnguyenthe333 on 23-09-2015 - 22:05 trong Chuyên đề toán THCS

Bài 150: Một chiếc thuyền chuyển động ngược dòng nước từ $A$ với $v_1=3km/h$.Một cano chuyển động từ $B$ với $v_2=10km/h$.Sau khi đi một đoạn, cano quay lại và ngược dòng về $B$.Khi thuyền đi từ $A$ đến $B$ thì cano đi được 4 lần quãng đường đó và về $B$ cùng lúc với thuyền.Biết thuyền và cano khởi hành cùng một lúc.Tính vận tốc dòng nước



#590487 $\boxed{{Topic}}$ Ôn thi học sinh giỏi lớp...

Đã gửi bởi Minhnguyenthe333 on 23-09-2015 - 18:18 trong Chuyên đề toán THCS

Cũng lâu rồi chưa đăng bài ở đây,mình xin đóng góp 1 số bài toán lấy từ 1 số nguồn(không có bài  hình)

Chúc TOPIC có nhiều bài đăng hay  :namtay

Bài 149:1) Cho $a, b, c$ là $3$ số không âm thỏa mãn điều kiện : $a^2+b^2+c^2 \leq 2(ab+bc+ca)$ (1) . Chứng minh bất đẳng thức :$(a+b+c) \leq 2( \sqrt{ab} + \sqrt{bc} + \sqrt{ca} ) $ (2)

Hỏi từ (2) có thể suy ra (1) hay không ? Vì sao ?

Từ giả thiết ta có: $(a+b+c)^2\leq 4(ab+bc+ca)$

BĐT$\Leftrightarrow  4(ab+bc+ca)\leq 4(\sqrt{ab}+\sqrt{bc}+\sqrt{ca})^2=4(ab+bc+ca)+8\sqrt{abc}(\sqrt{a}+\sqrt{b}+\sqrt{c})$

$\Leftrightarrow 8\sqrt{abc}(\sqrt{a}+\sqrt{b}+\sqrt{c})\geq 0$ (do $a,b,c\geq 0$)

Từ (2) không thể suy ra (1) vì $(a,b,c)=(16,1,1)$ thỏa (2) nhưng không thỏa (1)




#594083 $\boxed{{Topic}}$ Ôn thi học sinh giỏi lớp...

Đã gửi bởi Minhnguyenthe333 on 17-10-2015 - 17:49 trong Chuyên đề toán THCS

Bài trên sửa đi nha + có bài này khó quá mình hỏi luôn

197) 

b) Cho $\begin{Bmatrix} x^{2}+y^{2}=1 & \\\frac{x^{4}}{a}+\frac{y^{4}}{b} =\frac{1}{a+b} & \end{Bmatrix}$

CM

$\frac{x^{2006}}{a^{1003}}+\frac{y^{2006}}{b^{1003}}=\frac{2}{(a+b)^{1003}}$

 

Từ giả thiết áp dụng bđt Cauchy-Schwarz: $VT\geq \frac{(x^2+y^2)^2}{a+b}=\frac{1}{a+b}=VP$

Dấu "=" xảy ra nên $\frac{x^2}{a}=\frac{y^2}{b}=\frac{1}{a+b}$

 

hoặc biến đổi $1=(x^2+y^2)^2$ (nếu đề bài cho $a,b<0$) thì cũng suy ra như trên

$\Leftrightarrow \frac{x^{2006}}{a^{1003}}=\frac{y^{2006}}{b^{1003}}=\frac{1}{(a+b)^{1003}}$
$\Rightarrow  \frac{x^{2006}}{a^{1003}}+\frac{y^{2006}}{b^{1003}}=\frac{2}{(a+b)^{1003}}$




#594089 $\boxed{{Topic}}$ Ôn thi học sinh giỏi lớp...

Đã gửi bởi Minhnguyenthe333 on 17-10-2015 - 17:57 trong Chuyên đề toán THCS

Hình như bạn bị nhầm rồi.Bất đẳng thức Cauchy-Swarchz dạng Engel này chỉ được sử dụng khi mẫu lớn hơn 0.Một là bạn sai.Hai là ngừoi ra đề sai :D

mình có cách khác là biến đổi $1=(x^2+y^2)^2$




#603177 $\boxed{{Topic}}$ Ôn thi học sinh giỏi lớp...

Đã gửi bởi Minhnguyenthe333 on 14-12-2015 - 19:32 trong Chuyên đề toán THCS

áp dụng bđt cô-si có :   x  $\leq \frac{1 + x^2}{2}$

                                     y  $\leq \frac{1 + y^2}{2}$

   => 6x $\leq \frac{6 + 6x^2}{2}$

        7y $\leq \frac{7 + 7x^2}{2}$

   => A $\leq \frac{13 + x^2 + y^2}{2}$ 

max A = 13 với x = y = 1 (t/m)

$x,y$ có thể là số âm mà bạn




#584314 $\boxed{{Topic}}$ Ôn thi học sinh giỏi lớp...

Đã gửi bởi Minhnguyenthe333 on 23-08-2015 - 13:59 trong Chuyên đề toán THCS

Mình lập ra $\boxed{{Topic}}$ nhằm mục đích ôn đội tuyển đi thi học sinh giỏi lớp 9(đề nghị mấy anh/ chị sinh năm dưới 2001 không làm nha :lol:)
Vì lập $\boxed{{Topic}}$ lần đầu nên lời lẽ nó không hay lắm mong mọi người thông cảm nha.
Trước hết là ôn lại 1 vài bài lớp 8(ai có bài nào thì post hết vào nha mình không có nhiều bài lắm đâu
Bài 3: Một người đi xe máy trên quãng đường có chiều dài $S$ km. Trong nửa thời gian đầu, người đó đi đoạn đường $S_{1}$ với vận tốc $40km/h$. Trên đoạn đường còn lại, người đó đi nửa quãng đường đầu với vận tốc $V_{2}=80km/h$ và trong nửa quãng đường cuối đi với vận tốc $V_{3}$. Biết vận tốc trung bình trên cả quãng đường $S$ là $60km/h$. Tính $V_{3}$

Ta có phương trình: $S_1+S_2=S\Leftrightarrow \frac{t}{2}.40+S_2=60t\Rightarrow S_2=40t$ (1)
Lại có: $\frac{S_2}{2.v_3}+\frac{S_2}{2.80}=\frac{t}{2}\Rightarrow t=S_2(\frac{1}{v_3}+\frac{1}{80})$ (2)
Từ (1),(2) suy ra: $v_3=80km/h$



#594087 $\boxed{{Topic}}$ Ôn thi học sinh giỏi lớp...

Đã gửi bởi Minhnguyenthe333 on 17-10-2015 - 17:53 trong Chuyên đề toán THCS

không hiểu ??

thì 2 vế lũy thừa 1003 lên, nhưng sửa $a$ thành $b$ ở $y$




#590309 $\boxed{{Topic}}$ Ôn thi học sinh giỏi lớp...

Đã gửi bởi Minhnguyenthe333 on 22-09-2015 - 19:18 trong Chuyên đề toán THCS

Thấy topic buồn quá nên đăng thêm bài tập  :closedeyes:  :icon6:

Bài 142:
Chứng minh rằng với a,b,c>0 :

$\frac{1}{a+b}+\frac{1}{b+c}+\frac{1}{c+a}+\frac{1}{2\sqrt[3]{abc}}$ $\geq \frac{(a+b+c+\sqrt[3]{abc})^{2}}{(a+b)(b+c)(c+a)}$

$VT=\frac{c^2}{c^2(a+b)}+\frac{a^2}{a^2(b+c)}+\frac{b^2}{b^2(a+c)}+\frac{(\sqrt[3]{abc})^2}{2abc}\geq \frac{(a+b+c+\sqrt[3]{abc})^2}{b^2(a+c)+a^2(b+c)+c^2(a+b)+2abc}=\frac{(a+b+c+\sqrt[3]{abc})^2}{(a+b)(b+c)(c+a)}$

Dấu "=" xảy ra khi $a=b=c$




#584752 $\boxed{{Topic}}$ Ôn thi học sinh giỏi lớp...

Đã gửi bởi Minhnguyenthe333 on 24-08-2015 - 21:49 trong Chuyên đề toán THCS

Ủng hộ $Topic$ bài sau nhé.
Cho $PT$ sau $\frac{mx^{2}+(m-3)x+2m-1}{x+3}=0$. Xác định $m$ để $PT$ trên có $2$ nghiệm phân biệt $x_{1},x_{2}$ thoả mãn $21x_{1}+7m(2+x_{2}+x_{2}^{2})=48$.

$\Delta >0\Leftrightarrow (m-3)^2-4(2m^2-m)=-7m^2-2m+9=(m-1)(7m+9)>0\Rightarrow m>1$
Ta có: $21x_{1}+7m(2+x_{2}+x_{2}^{2})=7(3x_1+2m+mx_2+mx_2^2)=7(3x_1+2m+mx_2-mx_2+3x_2-2m+1)=21(x_1+x_2)+7=48$
Áp dụng hệ thức Viet suy ra $m$



#584398 $\boxed{{Topic}}$ Ôn thi học sinh giỏi lớp...

Đã gửi bởi Minhnguyenthe333 on 23-08-2015 - 18:14 trong Chuyên đề toán THCS

Thêm 2 bài nè bạn

Bài 12:Tìm tất cả các cặp số tự nhiên $x,y$ sao cho: $\sqrt{x}+\sqrt{y}=\sqrt{1989}$

Bài 13:Tìm $n$ ($n\in \mathbb{N}$) sao cho $n+t(n)+t(t(n))=60$, $t(n)$ là tổng các chữ số của $n$

Ví dụ: $n=1990$ thì $t(n)=1+9+9+0=19$




#584345 $\boxed{{Topic}}$ Ôn thi học sinh giỏi lớp...

Đã gửi bởi Minhnguyenthe333 on 23-08-2015 - 15:09 trong Chuyên đề toán THCS

Mình thấy bài này khá hay và nhẹ nhàng: 
Bài 10: Chứng minh rằng không thể tồn tại ba số nguyên dương $a,b,c$ thỏa mãn đồng thời cả ba bất đẳng thức: 
$a+\frac{1}{b}<3$ ; $b+\frac{4}{c}<4$ ; $c+\frac{9}{a}<5$
( Trích Đề khảo sát câu lạc bộ Học sinh giỏi môn Toán lớp 9 năm học 2015-2016)

Giả sử tồn tại 3 số $a,b,c$ thoả mãn đề bài.Khi đó ta có:
$a+b+c+\frac{1}{b}+\frac{4}{c}+\frac{9}{a}<12$
Lại có: $a+b+c+\frac{1}{b}+\frac{4}{c}+\frac{9}{a}\geq a+b+c+\frac{6^2}{a+b+c}\geq 2.\sqrt{36}=12$ (mâu thuẫn với đề bài)
Vậy ta có đpcm



#584334 $\boxed{{Topic}}$ Ôn thi học sinh giỏi lớp...

Đã gửi bởi Minhnguyenthe333 on 23-08-2015 - 14:53 trong Chuyên đề toán THCS

Bài 9:
1.Giải phương trình: $\frac{\left | 3-2x \right |-\left | x\right |}{\left | 2+3x \right |+x-2}=5$
2.Tìm $x,y\in \mathbb{Z}$ thoả $x^4+4y^4$ là số nguyên tố
(Trích đề thi HSG Toán 9 năm 1980-1981 vòng 2)



#587966 $\boxed{{Topic}}$ Ôn thi học sinh giỏi lớp...

Đã gửi bởi Minhnguyenthe333 on 08-09-2015 - 20:56 trong Chuyên đề toán THCS

Bài 125: Với $x,y$ là những số thực thả mãn đẳng thức $x^{2}y^{2}+2y+1=0,$ tìm giá trị lớn nhất và nhỏ nhất của biểu thức $P=\frac{xy}{3y+1}$

 

Ta có: $P^2=\frac{x^2y^2}{(3y+1)^2}=\frac{-2y-1}{(3y+1)^2}\Leftrightarrow y^2.9P^2+y(6P^2+2)+P^2+1=0$

Phương trình có nghiệm$\Leftrightarrow \Delta=(6P^2+2)^2-36P^2(P^2+1)\geq 0\Leftrightarrow 4\geq 36P^2-24P^2=12P^2$

$\Rightarrow P^2\leq \frac{1}{3}\Leftrightarrow -\frac{1}{\sqrt{3}}\leq P\leq \frac{1}{\sqrt{3}}$




#585959 $\boxed{{Topic}}$ Ôn thi học sinh giỏi lớp...

Đã gửi bởi Minhnguyenthe333 on 30-08-2015 - 09:19 trong Chuyên đề toán THCS

Bài 66:Cho $a,b,c> 0$. Chứng minh rằng
$\frac{a^{3}}{a^{2}+ab+b^{2}}+\frac{b^{3}}{b^{2}+bc+c^{2}}+\frac{c^{3}}{c^{2}+ac+c^{2}}\geq \frac{a+b+c}{3}$
Bài 67:Cho $a,b,c,d> 0$. Chứng minh rằng$\frac{a+c}{a+b}+\frac{b+d}{b+c}+\frac{c+a}{c+d}+\frac{d+b}{d+a}\geq 4$

Bài 67:
BĐT$\Leftrightarrow (a+c)(\frac{1}{a+b}+\frac{1}{c+d})+(b+d)(\frac{1}{b+c}+\frac{1}{d+a})\geq \frac{4(a+b+c+d)}{a+b+c+d}=4$
Dấu "=" xảy ra khi $a=b=c=d$



#587496 $\boxed{{Topic}}$ Ôn thi học sinh giỏi lớp...

Đã gửi bởi Minhnguyenthe333 on 05-09-2015 - 21:29 trong Chuyên đề toán THCS

Bài 120: Cho $x,y,z\geq 0$ thỏa $x+y+z=1$.Chứng minh rằng:

      $x+2y+z\geq 4(1-x)(1-y)(1-z)$




#586045 $\boxed{{Topic}}$ Ôn thi học sinh giỏi lớp...

Đã gửi bởi Minhnguyenthe333 on 30-08-2015 - 11:37 trong Chuyên đề toán THCS

Bài 69:Tìm giá trị lớn nhất của biểu thức $P:\left | \sqrt{x^2-4x+5}-\sqrt{x^2+6x+13} \right |$

$\Leftrightarrow P=\sqrt{(x-2)^2+1}+\sqrt{(x+3)^2+4}$

Xét các điểm $A(2,1),B(-3,2),C(x,0)$, ta có:$\left\{\begin{matrix} AB=\sqrt{(-3-2)^2+(2-1)^2}=26\\BC=\sqrt{(x+3)^2+4}\\ AC=\sqrt{(x-2)^2+1}\end{matrix}\right.$

Áp dụng bđt tam giác có: $P=\left | AC-BC\right |\leq AB=\sqrt{26}$
Dấu "=" xảy ra khi $A,B,C$ thẳng hàng $\Leftrightarrow \frac{x+3}{x+2}=\frac{-1}{-2}\Rightarrow x=7$



#587479 $\boxed{{Topic}}$ Ôn thi học sinh giỏi lớp...

Đã gửi bởi Minhnguyenthe333 on 05-09-2015 - 20:59 trong Chuyên đề toán THCS

Bài 119: Tìm tất cả các số tự nhiên $n$ biết $n$ có hai chữ số và $n$ chia hết cho tích các chữ số của nó

Đặt $n=\overline{xy}(1\leq x\leq 9,0\leq y\leq 9)$
Ta có: $n\vdots xy\Leftrightarrow \frac{10x+y}{xy}=\frac{10}{y}+\frac{1}{x}\in \mathbb{Z^+}$
Theo điều kiện của $x,y$ ta có: $(y,x)=(1,1);(2,1);(5,1)\Rightarrow n=(11;12;15)$



#587235 $\boxed{{Topic}}$ Ôn thi học sinh giỏi lớp...

Đã gửi bởi Minhnguyenthe333 on 04-09-2015 - 19:57 trong Chuyên đề toán THCS

Bài 104:Cho $a,b,c>0$ thỏa $abc=1$.Chứng minh

$\frac{1}{a^3+b^3+1}+\frac{1}{b^3+c^3+1}+\frac{1}{c^3+a^3+1}\leq 1$

P/S:các bạn còn cách nào khác ngoài dùng bdt $a^3+b^3\geq ab(a+b)$ không?




#587143 $\boxed{{Topic}}$ Ôn thi học sinh giỏi lớp...

Đã gửi bởi Minhnguyenthe333 on 04-09-2015 - 10:36 trong Chuyên đề toán THCS

Mình chỉ mới tìm được max thôi  :huh:

Từ gt có : $A^{2}=x^{2}.(x^{2}-6)^{2}=\frac{1}{2}\left [ 2x^{2}.(6-x^{2}).(6-x^{2}) \right ]$

Áp dụng BĐT Cauchy ta có : $2x^{2}.(6-x^{2}).(6-x^{2})\leq (\frac{2x^{2}+6-x^{2}+6-x^{2}}{3})^{3}$ $= 64$

=> $A^{2}\leq 32$

=>$A\leq 4\sqrt{2}$

Dấu ''='' xảy ra <=> $2x^{2}=6-x^{2}=6-x^{2}$

                        <=> $x=\sqrt{2}$

Theo mình thì:

Max: $A\leq 3(3^2-6)=9$ khi $x=3$

Còn $A_{min}=-4\sqrt{2}$ khi $x=\sqrt{2}$




#613644 Topic về phương trình và hệ phương trình

Đã gửi bởi Minhnguyenthe333 on 08-02-2016 - 14:35 trong Phương trình - hệ phương trình - bất phương trình

Bài 203: Giải phương trình:

                $\frac{|x|\sqrt{x^2+1}-x^2-3+2\sqrt{2}}{|x|\sqrt{x^2+1}+x^2+3-2\sqrt{2}}=x^2$




#612715 Topic về phương trình và hệ phương trình

Đã gửi bởi Minhnguyenthe333 on 03-02-2016 - 18:02 trong Phương trình - hệ phương trình - bất phương trình

Thứ nhất: sau khi mình check thì đề có chút nhầm lẫn:

Lại là:

Bài 176: Tìm $x\in \left [ 0;3 \right ]$ thỏa mãn:

$x\sqrt{5-x}+\left ( 3-x \right )\sqrt{2+x}=\frac{3\sqrt{7}}{\sqrt{2}}$

Thứ hai: Lời giải của bác cũng có nhầm lẫn:

$18-6x(x-3)<(\frac{63}{2})^2 (?????)$ lưu ý điều kiện $x\in \left [ 0;3 \right ]$

P/S: $x=\frac{3}{2}$ là Max xảy ra

P/S lần nữa: Mời mọi người! Xin hết ~!

$VT^2=18-6x(x-3)\leqslant \frac{63}{2}<=>-(2x-3)^2\leqslant 0$ (luôn đúng)
Dấu "=" xảy ra nên $x=\frac{3}{2}$